Lema de Thue

Vamos começar enunciando o Lema de Thue, prová-lo, e em seguida apresentar problemas resolvidos e propostos para o leitor.

Lema de Thue: Seja n data-recalc-dims=1" /> um inteiro e a um inteiro tal que mdc(a,n)=1. Então existem x,y tais que 0<|x|,|y|\le \lfloor\sqrt{n}\rfloor e

ax\equiv y\:(mod\:n)

Prova: Tome primeiro o caso em que n não é quadrado perfeito. Considere todos os pares ordenados de inteiros (x,y) com ambas as variáveis entre 0 e \lfloor\sqrt{n}\rfloor. Logo, há (\lfloor\sqrt n\rfloor+1)^2=\lfloor\sqrt n\rfloor^2+2\cdot\lfloor\sqrt n\rfloor+1 data-recalc-dims=(\sqrt n-1)^2+2(\sqrt n-1)+1=n-2\sqrt n+1+2\sqrt n-2+1=n" /> pares, ou seja, há pelo menos n+1 pares (x,y), e como a expressão ax-y só pode assumir n valores módulo n, segue por PCP que existem dois pares diferentes (x_1,y_1) e (x_2,y_2) tais que ax_1-y_1\equiv ax_2-y_2\:(mod\:n) \iff a(x_1-x_2)\equiv (y_1-y_2)\:(mod\:n). Mas note que, como a e n são primos entre si, x_1=x_2 \iff y_1=y_2,, e qualquer das igualdades geraria igualdade dos pares, um absurdo, e logo |x_1-x_2|,|y_1-y_2| data-recalc-dims=0" />. Por outro lado, |x_1-x_2|\le max\{x_1,x_2\}\le \lfloor\sqrt{n}\rfloor e |y_1-y_2|\le max\{x_1,x_2\}\le \lfloor\sqrt{n}\rfloor, e tomando x=x_1-x_2 e y=y_1-y_2 segue o que queríamos.

Há também o:

Lema de Thue generalizado: Sejam \alpha e \beta dois números reais positivos tais que \alpha\beta\ge p. Então para um inteiro a coprimo com p, existem x,y inteiros tais que 0<x\le\alpha, 0<y\le\beta e

ax\equiv y\:(mod\:p)

A prova é análoga e deixada como exercício.

Vamos aos exemplos resolvidos!

Exemplo 1: Seja p um primo para o qual existe um inteiro positivo a tal que p divide 2a^2-1. Prove que existem inteiros b e c de modo que p=2b^2-c^2

Solução: Pelo Lema de Thue, como a é coprimo com p, tome b,c inteiros tais que 0<|b|,|c|<\sqrt{p} e ca\equiv b\:(mod\:p). Veja que pela limitação de b e c, ambos são primos com p e portanto invertíveis módulo p. Logo,2a^2-1\equiv 0 \:(mod\:p)\iff 2(b\cdot c^{-1})^2-1\equiv 0\:(mod\:p) \iff 2b^2-c^2\equiv 0\:(mod\:p). Agora, note que 2b^2-c^2 data-recalc-dims=-c^2>-(\sqrt{p})^2=-p" /> e 2b^2-c^2<2b^2<2p e como p|2b^2-c^2, há duas possibilidades: 2b^2-c^2=0\Rightarrow \sqrt 2=\dfrac{c}{b}\Rightarrow \sqrt2 é racional, um absurdo, ou 2b^2-c^2=p, que é a única que pode ocorrer, demonstrando a existência de b e c nas condições do enunciado que queríamos.

 

Exemplo 2: Se p é um primo da forma 3k+1, então existem inteiros a e b tais que p=a^2+ab+b^2

Solução:  Inicialmente, vamos provar que -3 é resíduo quadrático módulo p. Note que \left(\frac{-3}{p}\right)=\left(\frac{-1}{p}\right)\left(\frac{3}{p}\right), mas pela Lei da Reciprocidade Quadrática temos que \left(\frac{3}{p}\right)=(-1)^{\frac{p-1}{2}\frac{3-1}{2}}\cdot \left(\frac{p}{3}\right) , e pelo Critério de Euler \left(\frac{-1}{p}\right)=(-1)^{\frac{p-1}{2}}, portanto: \left(\frac{-3}{p}\right)=\left(\frac{-1}{p}\right)\left(\frac{3}{p}\right)=\left(\frac{-1}{p}\right)(-1)^{\frac{p-1}{2}\frac{3-1}{2}}\left(\frac{p}{3}\right)=(-1)^{p-1}\left(\frac{1}{p}\right)=1 e segue o que foi afirmado. Seja agora m tal que m^2\equiv-3\:(mod\:p). Como p é ímpar, 2 é invertível, e tome x\equiv (m-1)\cdot 2^{-1}\:(mod\:p)\Rightarrow (2x+1)^2\equiv -3\:(mod\:p)\Rightarrow 4x^2+4x+4\equiv 0\:(mod \:p)\Rightarrow x^2+x+1\equiv 0\:(mod\:p).

Pelo Lema de Thue, como x é coprimo com  p, tome 0<|a|,|b|<\sqrt{p} tais que ax\equiv b\:(mod\:p) \iff x\equiv b\cdot a^{-1} \:(mod\:p). Então podemos substituir o valor de x e obter x^2+x+1\equiv 0\:(mod\:p)\Rightarrow (b\cdot a^{-1})^2+b\cdot a^{-1}+1\equiv 0\:(mod\:p)\iff a^2+ab+b^2\equiv 0\:(mod\:p) . Agora, veja que a^2+ab+b^2 data-recalc-dims=0" /> pois, de fato, se ab data-recalc-dims=0" /> o resultado é claro, e se ab<0, a^2+ab+b^2 data-recalc-dims=a^2+2ab+b^2=(a+b)^2>0" />. Por outro lado, pela limitação de a,b do Lema de Thue, sabe-se que a^2+ab+b^2<3(\sqrt{p})^2=3p e como p|a^2+ab+b^2, essa expressão só pode assumir os valores p e 2p. Mas se tivéssemos a^2+ab+b^2=2p\Rightarrow a^2+ab+b^2\equiv 2\:(mod\:3). Se a\equiv0, b^2\equiv2, um absurdo. Se a\equiv1, b^2+b\equiv1, um absurdo pois x^2+x\equiv 0,2,0 ao fazermos x=0,1,2. Logo, a\equiv 2\Rightarrow b^2+2b\equiv 1, também absurdo pois x^2+2x\equiv 0,0,2 com x=0,1,2. Finalmente, só há uma possibilidade, que é a^2+ab+b^2=p, como queríamos.

 

Teorema: Um número inteiro n pode ser escrito como soma de dois quadrados perfeitos se,e somente se, sua fatoração em primos não contém nenhum primo da forma 4k+3 com expoente ímpar.

Prova: Comecemos pelo seguinte:

Lema 1: Se p é um primo da forma 4k+3 e p|x^2+y^2 então p|x e p|y, donde p^2|x^2+y^2.

Prova: p|x^2+y^2 \iff x^2\equiv -y^2\:(mod\:p) \iff (x^2)^{\frac{p-1}{2}}\equiv(-y^2)^{\frac{p-1}{2}}\:(mod\:p), mas pelo Pequeno Teorema de Fermat, supondo que p não divide x nem y, vale que x^{p-1}\equiv y^{p-1}\equiv 1\:(mod\:p) e logo (-1)^{\frac{p-1}{2}}\equiv 1\:(mod\:) \iff (-1)^{2k+1}\equiv 1 \:(mod\:p) \iff -1\equiv 1\:(mod\:p), um absurdo. Logo, p divide um dentre x e y e logo divide o outro, e segue p|x e p|y.

Lema 2: Se p é um primo da forma 4k+1, então ele pode ser escrito como soma de quadrados.

Prova: Pelo critério de Euler, \left(\dfrac{-1}{p}\right)\equiv (-1)^{\frac{p-1}{2}}=(-1)^{2k}=1\:(mod\:p)\Rightarrow -1 é resíduo quadrático módulo p, ou seja, existe x tal que x^2\equiv-1. Tome, pelo Lema de Thue, já que mdc(x,p)=1, existem inteiros a,b tais que 0<|a|,|b|<\sqrt p e ax\equiv b\:(mod\:p). Então x\equiv b\cdot a^{-1} \:(mod\:p)\iff (b\cdot a^{-1})^2\equiv -1\:(mod\:p) \iff b^2\equiv -a^2 \:(mod\:p) \iff p|a^2+b^2, e como nós sabemos que 0<a^2+b^2<2(\sqrt{p})^2=2p, segue que a única possibilidade é a^2+b^2=p.

Agora, veja que se dois números podem ser escritos como soma de quadrados, então seu produto também pode, pois: (d^2+e^2)(f^2+g^2)=(df)^2+2defg+(eg)^2+(dg)^2-2defg+(ef)^2=(df+eg)^2+(dg-ef)^2, logo, como 2=1^2+1^2 e todo primo da forma 4k+1 é escrito como soma de dois quadrados pelo Lema 2, basta que o número tenha expoentes pares nos primos da forma 4k+3, de modo que possamos os escrever como  q^2=q^2+0^2 e utilizar a propriedade citada acima. Além disso, se p=4k+3 é um primo que divide n=a^2+b^2 então pelo Lema 1 vale que p|a e p|b, donde \dfrac{n}{p^2}=\left(\dfrac{a}{p}\right)^2+\left(\dfrac{b}{p}\right)^2. Se p ainda divide \dfrac{n}{p^2}, podemos realizar raciocínio análogo, e ir retirando duas unidades do expoente de p em n. Caso esse fosse ímpar, em algum momento do algoritmo teríamos o p com expoente 1, e pelo lema 1 seguiria um absurdo. Logo, a condição dos expoentes dos fatores primos da forma 4k+3 de um número serem pares é necessária e suficiente para que ele seja escrito como soma de quadrados, como queríamos.

 

Problemas Propostos:

Problema 1: Seja a um inteiro positivo e p um divisor primo de a^3-3a+1, com p\neq3. Prove que p é da forma 9k+1 ou 9k-1, onde k é um inteiro.

Obs: A solução desse problema pode ser encontrada em OBM-2017-P6.

Problema 2: Seja p um número primo. Prove que existem inteiros x,y tais que p=2x^2+3y^2 se, e somente se, p\equiv 5,11\:(mod\:24).

Problema 3: Seja S o conjunto de todos os inteiros positivos que podem ser repreentados como a^2+5b^2 para inteiros a,b tais que mdc(a,b)=1. Seja p um número primo tal que p=4n+3 para algum inteiro positivo n. Mostre que se para algum inteiro positivo k o número kp pertence à S, então 2p também está em S.

Problema 4: Prove que a equação x^3-x+9=5y^2 não tem solução nos inteiros.